Rotations in Bloch Sphere about an arbitrary axis

In summary, the conversation discusses the interpretation of rotations on a single qubit state represented by a Bloch vector. It is explained that the rotation operators can be written in terms of Pauli matrices and that the rotation matrix for a given axis can be derived using a Taylor expansion. The question then arises as to how to prove that this rotation is indeed about the given axis. It is suggested that a proof by demonstration can be used, by comparing the effect of the rotation on two different vectors and showing that it is consistent with a rotation about the given axis.
  • #1
polyChron
2
0
Hey,
(I have already asked the question at http://physics.stackexchange.com/questions/244586/bloch-sphere-interpretation-of-rotations, I am not sure this forum's etiquette allows that!)

I am trying to understand the following statement. "Suppose a single qubit has a state represented by the Bloch vector ##\vec{\lambda}##. Then the effect of the rotation ##R_{\hat{n}}(\theta)## on the state is to rotate it by an angle $\theta$ about the ##\hat{n}## axis of the Bloch sphere. This fact explains the rather mysterious looking factor of two in the definition of the rotation matrices."
I could work out that the rotation operators ##R_x(\theta)##, ##R_y(\theta)## and ##R_z(\theta)## are infact rotations about the ##X,Y## and ##Z## axis. But how do I extend this for ##R_{\hat{n}}(\theta)## and prove the above statement. Please point me in the right direction.

Thanks.
 
Physics news on Phys.org
  • #2
I think what you want is what the pauli spin observable is along the "[itex]\hat{n}[/itex]" axis.
Where [itex]\hat{n}[/itex] is a unit vector with components [itex](n_{x},n_{y},n_{z})[/itex],

[itex]\sigma_{\hat{n}} = \sigma_{x}n_{x} + \sigma_{y} n_{y} + \sigma_{z} n_{z}[/itex].

If you know what [itex]R_{x}(\theta)[/itex] is in terms of [itex]\sigma_{x}[/itex], then I think you can work out what [itex]R_{\hat{n}}(\theta)[/itex] is in terms of [itex]\sigma_{\hat{n}}[/itex].
 
  • #3
jfizzix said:
I think what you want is what the pauli spin observable is along the "[itex]\hat{n}[/itex]" axis.
Where [itex]\hat{n}[/itex] is a unit vector with components [itex](n_{x},n_{y},n_{z})[/itex],

[itex]\sigma_{\hat{n}} = \sigma_{x}n_{x} + \sigma_{y} n_{y} + \sigma_{z} n_{z}[/itex].

If you know what [itex]R_{x}(\theta)[/itex] is in terms of [itex]\sigma_{x}[/itex], then I think you can work out what [itex]R_{\hat{n}}(\theta)[/itex] is in terms of [itex]\sigma_{\hat{n}}[/itex].

Thanks for your reply.
Yes I have figured that out as well. Let me explain.
I know the rotation matrices in terms of the Pauli matrices, i.e [itex]R_x(\theta) = e^{-i\sigma_x /2}[/itex] and the rotation matrices for [itex]\sigma_y[/itex] and [itex]\sigma_z[/itex] follows in the same manner. I could also prove that [itex]R_{\hat{n}}(\theta) = cos(\frac{\theta}{2})I - i sin(\frac{\theta}{2})(n_x\sigma_x+n_y\sigma_y+n_z\sigma_z)[/itex] using the Taylor expansion. But the difficulties for me start from here. How do I show that [itex]R_{\hat{n}}(\theta)[/itex] is infact a rotation about [itex]\hat{n}[/itex] axis by [itex]\theta[/itex]. How can I construct a concrete proof?
 
  • #4
polyChron said:
Thanks for your reply.
Yes I have figured that out as well. Let me explain.
I know the rotation matrices in terms of the Pauli matrices, i.e [itex]R_x(\theta) = e^{-i\sigma_x /2}[/itex] and the rotation matrices for [itex]\sigma_y[/itex] and [itex]\sigma_z[/itex] follows in the same manner. I could also prove that [itex]R_{\hat{n}}(\theta) = cos(\frac{\theta}{2})I - i sin(\frac{\theta}{2})(n_x\sigma_x+n_y\sigma_y+n_z\sigma_z)[/itex] using the Taylor expansion. But the difficulties for me start from here. How do I show that [itex]R_{\hat{n}}(\theta)[/itex] is infact a rotation about [itex]\hat{n}[/itex] axis by [itex]\theta[/itex]. How can I construct a concrete proof?

I think what you should do is a proof by demonstration. Compare a bloch vector before [itex]\hat{u}[/itex] and after [itex]\hat{u}'[/itex] a rotation about the n-axis.
Both [itex]\hat{u}[/itex] and [itex]\hat{u}'[/itex] dotted with [itex]\hat{n}[/itex] should give the same value, and the vectors themselves should have the same magnitude. This proves that [itex]R_{n}(\theta)[/itex] is at least some sort of rotation about the [itex]\hat{n}[/itex]-axis.

To find the angle, you need to project [itex]\hat{u}[/itex] and [itex]\hat{u}'[/itex] onto the plane perpendicular to [itex]\hat{n}[/itex]. The dot product of these projected vectors will be the magnitude of each times the cosine of the angle between them, and hopefully that angle will be none other than [itex]\theta[/itex].
 

1. What is the Bloch sphere and how does it relate to quantum mechanics?

The Bloch sphere is a geometric representation of a qubit in quantum mechanics. It is a unit sphere with the north and south poles representing the two basis states of a qubit, and all other points on the sphere representing superpositions of those states. It is used to visualize the rotation and manipulation of qubits in quantum information processing.

2. What is a rotation in the Bloch sphere?

A rotation in the Bloch sphere is a unitary transformation that represents a change in the state of a qubit. It involves rotating the state vector of the qubit around the surface of the sphere, while keeping the length of the vector constant. Rotations can be performed about any axis on the sphere, allowing for a wide range of transformations.

3. How is a rotation about an arbitrary axis on the Bloch sphere performed?

A rotation about an arbitrary axis on the Bloch sphere can be performed using the Euler-Rodrigues formula. This involves decomposing the rotation into three sequential rotations about the x, y, and z axes, and then applying each rotation successively to the qubit state vector. The resulting state vector will be rotated about the desired arbitrary axis on the Bloch sphere.

4. What is the significance of rotations in the Bloch sphere in quantum computing?

Rotations in the Bloch sphere are essential in quantum computing as they allow for the manipulation and control of qubits. By applying rotations, we can change the state of a qubit and perform operations such as unitary gates, entanglement, and measurement. Different rotations can also be combined to perform more complex transformations, making them a crucial tool in quantum algorithms.

5. Are rotations in the Bloch sphere reversible?

Yes, rotations in the Bloch sphere are reversible. Since they are unitary transformations, they preserve the length of the qubit state vector and can be reversed by applying the inverse rotation. This reversibility is important in quantum computing, as it allows for the ability to undo operations and maintain the coherence and integrity of the qubit state.

Similar threads

  • Quantum Physics
Replies
1
Views
226
  • Quantum Physics
Replies
4
Views
1K
Replies
4
Views
2K
Replies
4
Views
881
  • Atomic and Condensed Matter
Replies
1
Views
2K
  • Advanced Physics Homework Help
Replies
2
Views
2K
Replies
16
Views
1K
  • Linear and Abstract Algebra
Replies
6
Views
2K
  • Advanced Physics Homework Help
Replies
1
Views
1K
  • Introductory Physics Homework Help
Replies
7
Views
6K
Back
Top